I will mark you brainiest!

The sum of the interior angles of an octagon is:
A) 1080º.
B) 180º.
C) 360º.
D) 720º.

Answers

Answer 1

Answer:

A) 1080º

Step-by-step explanation:

I remember learning this in math class. The octagon has 8 interior angles and 8 exteriors angle. The sum of the interior angles of an octagon is 1080º

Answer 2

Answer:

The sum of the interior angles of an octagon can be found using the formula: (n-2) x 180 degrees, where n is the number of sides of the polygon.

For an octagon, n = 8, so the sum of the interior angles is (8-2) x 180 = 6 x 180 = 1080 degrees.

Therefore, the correct answer is A) 1080º.


Related Questions

The value, V, of a particular automobile (in dollars) depends on the number of miles, m, the car has been driven, according to the function V(m).

A. Interpret the statement, V(70, 000) = 12, 500, in the context of this application. Write a sentence that tells someone what that statement means and include the appropriate
units.

B. In the statement f '(70, 000) = − 15

i. What are the units of the 70, 000?

ii. What are the units of the − 15?

iii. Interpret the statement, f '(60, 000) = − 15, in the context of this application.
In other words, using everyday language, write a sentence that tells someone
what that statement means and include the appropriate units.

Answers

A. The statement V(70,000) = 12,500 means that the value of the automobile when it has been driven for 70,000 miles is $12,500.

B.

i. The units of 70,000 are miles, since it represents the number of miles the car has been driven.

ii. The units of -15 are dollars per mile, since it represents the rate of change of the value of the car with respect to the number of miles driven.

iii. The statement f'(60,000) = -15 means that when the car has been driven for 60,000 miles, the rate of change of its value is -$15 per mile. In other words, for every additional mile the car is driven after 60,000 miles, its value decreases by $15.

Trapezoid W X Y Z is rotated about point A 180 degrees clockwise to form trapezoid W prime X prime Y prime Z prime. Trapezoid W prime X prime Y prime Z prime is reflected across the line of reflection m to form trapezoid W double-prime X double-prime Y double-prime Z double-prime.
Analyze the diagram. What is the composition of transformations that was applied to map WXYZ to W''X''Y''Z''?

The first transformation was a


.



The second transformation was a

Answers

The first transformation was a rotation about point A.

The second transformation was a reflection across line M.

What is a rotation?

In Mathematics, a rotation can be defined as a type of transformation which moves every point of the object through a number of degrees around a given point, which can either be clockwise or counterclockwise (anticlockwise) direction.

By critically observing the diagram which illustrates the sequence of transformations, we can logically deduce that the first transformation was a clockwise rotation about point A by 180 degrees.

Furthermore, the second transformation that maps W'X'Y'Z' to W''X''Y''Z'' is a reflection across the line of reflection M.

Read more on transformation here: https://brainly.com/question/15832612

#SPJ1

APR 30-Year Term 20-Year Term 15-Year Term
5.5 $5.68 $6.88 $8.17
6.0 $6.00 $7.16 $8.44
6.5 $6.32 $7.46 $8.71
7.0 $6.65 $7.75 $8.99
7.5 $6.99 $8.06 $9.27


Determine the percent decrease in total principal and interest paid between a 30-year term mortgage and a 15-year mortgage with a principal balance of $242,300.00 and a 6.5% APR. Round the final answer to the nearest tenth. (4 points)

30.0%

31.1%

45.0%

45.1%

Answers

The percentage decrease in total principal and interest paid between a 30-year term mortgage and a 15-year term mortgage at an APR of 6.5% and principal of $242,300.00 is about 31.1%

What is a mortgage?

A mortgage is a loan used to purchase property.

The principal balance = $242,300.00

The Annual Percentage Rate, APR = 6.5%

The term of the loan = 30-year and 15-year

The monthly payment for the loan per each term can be obtained using the following formula;

M = P·[r·(1 + r)^n]/[(1 + r)^n - 1]

Where;

M = The monthly payment

P = The principal amount of the loan = $242,300.00

r = The monthly interest rate = APR/12 = 0.065/12

n = The the number of periods of payment = 12 × 30 = 360

The monthly payment for the 30-year loan term is therefore;

M₃₀ = 242,300 × [(0.065/12)·(1 + (0.065/12))^360]/[(1 + (0.065/12))^360 - 1] ≈ 1531.5

The total payment for the 30-year term ≈ 360 × 1531.5 = 551,340.3

The total payment for the 30-year term is about $551,340.3

The monthly payment for the 15-year loan term is found as follows;

n = 12 × 15 = 180

M₁₅ = 242,300 × [(0.065/12)·(1 + (0.065/12))^180]/[(1 + (0.065/12))^180 - 1] ≈ 2110.7

The total payment in 15 years ≈ 15 × 12 × 2110.7 = 379962

The total payment for the 15-year loan term is about $379,962

The percentage decrease, is therefore;

((551,340.3 - 379,962)/551,340.3) × 100 ≈ 31.1%

The percentage decrease is about 31.1%

Learn more on mortgage calculations here: https://brainly.com/question/28041674

#SPJ1

i’ve been struggling for hours and i still can’t get it!! someone please help

Answers

Answer:

40.1 cm

Step-by-step explanation:

For this problem we will use trigonometry. To find x, we have to remember SOH CAH TOA

Our angle is 55, and the we have the value adjacent to the angle which is 23 cm so we will take that and make an equation because we are missing the hypotenuse:

CAH = adjacent/hypotenuse

cos = ^

cos ( 55 ) = 23/x

multiply x both sides

xcos(55) = 23

divide cos(55) from left

x = 23/cos(55)

input in calculator (make sure in right mode degrees)

23/(cos(55))

x = 40.09927 cm

x = 40.1 cm

Choose a U.S. household at random and record the number of vehicles. The table below shows the probability model.
Define F as a household that owns four cars. Which probability is correct?

Answers

the correct probability is option c)[tex]P(F^c)[/tex] = 0.95.

What is the probability?

Probability refers to potential. A random event's occurrence is the subject of this area of mathematics. The value is stated from zero to one. Mathematics has incorporated probability to forecast the likelihood of various events. The degree to which something is likely to happen is basically what probability means. You will discover the likelihood of outcomes for a random experiment using this fundamental probability theory, which is also applied to probability distribution.

from the question:

According to the table, the likelihood that a household owns four cars is given by P(4) = 0.05.

Four automobiles in one family constitute the occurrence F. A home with more than four automobiles is the complement of this occurrence, [tex]F^c[/tex].

The total of all outcomes' probabilities must equal 1. Thus, by deducting P(4) from 1 we may derive P([tex]F^c[/tex]):

P([tex]F^c[/tex]) = 1 - P(4) = 1 - 0.05 = 0.95

Therefore, the correct probability is option c) P([tex]F^c[/tex]) = 0.95.

To know more about probability visit:

https://brainly.com/question/30034780

#SPJ1

Determine the length of x in the triangle. Give your answer to two decimal places. most importantly show the steps, please.

Answers

Answer:

x=43.85

Step-by-step explanation:

solution Given.

let the given angle be θ: 20 degree.

It's opposite side is 12.

It's hypotenuse is x.

Since

We have

Sin θ= opposite/hypotenuse

Sin20 degree=12/x

0.34202014332=12/x

doing criss cross multiplication

x=12/0.34202014332

therefore x=43.8570660032

in two decimal form x=43.85

How much money should be deposited today in an account that earns 2.5% compounded monthly so that it will
accumulate to $13,000 in 4 years?
i Click the icon to view some finance formulas.

Answers

Answer: We can use the formula for the future value of an annuity with monthly compounding to determine how much money should be deposited:

FV = P * ((1 + r/n)^(n*t) - 1) / (r/n)

where:

FV = future value (the amount we want to accumulate, which is $13,000 in this case)

P = periodic payment (the amount we need to deposit each month)

r = annual interest rate (2.5% in this case)

n = number of compounding periods per year (12 for monthly compounding)

t = number of years (4 in this case)

We want to solve for P, so we can rearrange the formula as follows:

P = FV * (r/n) / ((1 + r/n)^(n*t) - 1)

Substituting the given values, we get:

P = 13000 * (0.025/12) / ((1 + 0.025/12)^(12*4) - 1)

Calculating this expression gives us:

P = $279.27 (rounded to the nearest cent)

Therefore, we need to deposit $279.27 each month for 4 years in an account that earns 2.5% compounded monthly in order to accumulate $13,000. Alternatively, if we want to make a single deposit today, we can multiply this monthly amount by 12 and then by 4, which gives us:

P = $13,414.08 (rounded to the nearest cent)

Therefore, we need to deposit $13,414.08 today in an account that earns 2.5% compounded monthly in order to accumulate $13,000 after 4 years.

Step-by-step explanation:

Draw a unit circle for each of the following then find several positive and negative real
numbers t which determine a point Q with the given coordinates. Then write a formula
for t in terms of 2kπ.
1. ( 0, 1)

Answers

The fοrmula fοr t in terms οf 2kπ is: t = kπ + π/2, where k is an integer.

How tο draw a unit circle?

Tο draw a unit circle fοr the given cοοrdinates, we first draw the hοrizοntal and vertical axes intersecting at the οrigin (0,0):

Next, we draw a circle with radius 1 centered at the οrigin:

Tο find pοints οn the circle with y-cοοrdinate 1, we lοοk at the pοint where the circle intersects the vertical axis. This οccurs when x = 0, sο the pοint οn the circle with y-cοοrdinate 1 is (0,1):

(0,1)

Tο find οther pοints οn the circle, we can use the Pythagοrean identity:

sin²(t) + cοs²(t) = 1

Since we want y = sin(t) tο be 1, we can sοlve fοr x = cοs(t)

cοs(t) = sqrt(1 - sin²(t))

Using this fοrmula, we can find several pοsitive and negative real numbers t that determine pοints Q οn the circle with y-cοοrdinate 1:

t = 0 radians (0 degrees): Q = (1,0)

t = π/6 radians (30 degrees): Q = (√3/2, 1/2)

t = π/4 radians (45 degrees): Q = (√2/2, √2/2)

t = π/3 radians (60 degrees): Q = (1/2, √3/2)

t = π/2 radians (90 degrees): Q = (0,1)

t = 7π/6 radians (-150 degrees): Q = (-√3/2, 1/2)

t = 3π/4 radians (-135 degrees): Q = (-√2/2, √2/2)

t = 5π/6 radians (-120 degrees): Q = (-1/2, √3/2)

t = π radians (-180 degrees): Q = (-1,0)

Tο write a fοrmula fοr t in terms οf 2kπ, we can use the inverse sine functiοn:

sin(t) = 1

t = sin⁻¹(1) + 2kπ

Since sin(π/2) = 1, we have:

t = π/2 + 2kπ

Sο the fοrmula fοr t in terms οf 2kπ is:

t = kπ + π/2, where k is an integer.

To learn more about circles,

https://brainly.com/question/29142813

#SPJ1

Which equations can be used to find the lengths of the legs of the triangle? Select three options. 0.5(x)(x + 2) = 24 x(x + 2) = 24 x2 + 2x – 24 = 0 x2 + 2x – 48 = 0 x2 + (x + 2)2 = 100

Answers

None of the provided equations are direct formulas for finding the lengths of the legs of a triangle.

What is equation?

A mathematical definition of an equation is a claim that two expressions are equal when they are joined by the equals symbol ("="). For illustration, 2x - 5 Equals 13.

Here,

5 and 13 are phrases for 2x.

FROM THE QUESTION?

The number of x, which is one of the lengths of a right triangle's legs, can be determined by solving the quadratic equation [tex]0.5(x)(x+2)=24[/tex] . It is not a straightforward method, though, to determine the lengths of the legs.

It is possible to simplify and calculate the quadratic equation [tex]x(x+2)=24[/tex] to determine the values of x, which correspond to the lengths of a rectangle whose area is 24. Once more, there is no simple method for determining the lengths of a triangle's legs.

You can simplify and calculate the equation [tex]x^2+(x+2)2=100[/tex] to determine the value of x, which is one of the lengths of a right triangle's legs. It is not a straightforward method, though, to determine the lengths of the legs.

The lengths of a triangle's legs cannot be determined directly using any of the given formulae.

To know more about equation:

brainly.com/question/29657988

#SPJ1

Which side lengths form a right triangle? Choose all answers that apply: A 5, 6,√30 B 2.5, √18, 5 C √2, 2, √6​

Answers

Since 6 = 6, these side lengths do form a right triangle.

Therefore, the answer is C: √2, 2, √6.

How are lengths of sides of a triangle calculated?

The Pythagorean theorem, which asserts that in a right triangle, the square of the length of the hypotenuse (the side opposite the right angle) is equal to the sum of the squares of the lengths of the other two sides, can be used to identify which side lengths make up a right triangle.

A) The Pythagorean theorem enables us to deduce that: 52 + 62 = 25 + 36 = 61

[tex]\sqrt{30^{2}[/tex] = 30

These side lengths do not create a right triangle because 61 > 30.

B) The Pythagorean theorem enables us to observe that:

[tex]2.5^2 + \sqrt{18^{2} }[/tex] = 6.25 + 18 = 24.25

Also,

[tex]5^{2}[/tex] = 25

Since 24.25 < 25, a right triangle cannot be formed by these side lengths.

C) The Pythagorean theorem enables us to observe that:

[tex]\sqrt{2^{2} } + 2^2[/tex] = 2 + 4 = 6

[tex]\sqrt{6^{2} }[/tex] = 6

Learn more about triangles here:

brainly.com/question/2773823

#SPJ9

c is greater than or equal to 3

Answers

Answer:

Step-by-step explanation:

equal

Simplify 7/12 + 5/18 =

Answers

Answer:31/36

Step-by-step explanation:

we should find greatest common divisor(GCD) ,

GCD(12,18)=36

7/12=7*3/36=21/36

5/18=5*2/36=10/36

21/36+10/36=31/36

Answer:

31/36

Step-by-step explanation:

1) Find the Least Common Denominator (LCD) of [tex]\frac{7}{12}[/tex], [tex]\frac{5}{18}[/tex]. In other words, find the Least Common Multiple (LCM) of 12, 18.

Method 1: By Listing Multiples

1) List the multiples of each number

Multiples of 12: 12, 24, 36, ...

Multiples of 18: 18, 36, ...

2) Find the smallest number that is shared by all rows above. This is the LCM.

LCM = 36

Method 2: By Prime Factors

1) List the prime factors of each number.

Prime Factors of 12: 2, 2, 3

Prime Factors of 18: 2, 3, 3

2) Find the union of these primes.

2, 2, 3, 3

3) Multiply these number: 2 x 2 x 3 x 3 = 36. This is the LCM.

LCM = 36

STEP 2: Make the denominators the same as the LCD.

[tex]\frac{7\times3}{12\times3} +\frac{5\times2}{18\times2}[/tex]

3) Simplify. Denominators are now the same.

[tex]\frac{21}{36} +\frac{10}{36}[/tex]

4) Join the denominators.

[tex]\frac{21+10}{36}[/tex]

5) Simplify.

[tex]\frac{31}{36}[/tex]

Thank you,

Eddie.

I’m confused, how do I create a dot-plot with only one number in the tens?

Answers

The dot represents the data point of 5, and there are no other data points to include on the dot plot.

You can still make a dot plot even if you just have one number in the tens by placing a dot on a number line above the number. If your data point is 5, for instance, you can represent it by placing a dot above the tick mark for 5 on a number line that has tick marks for each integer from 0 to 9. Here is an illustration of how the dot plot might appear:

  |      

5  •      

  |      

  |      

  |      

  |      

  |      

  |      

  |      

  |      

  +   -   -    -   -   -   -   -

  0  1   2   3  4  5  6  7

 

There are no additional data points to place on the dot plot because the dot in this illustration represents the data point of 5.

To know more about Dot-Plot visit:

https://brainly.com/question/16269447

#SPJ1

Which expression gives the distance between the points (-11, 2) and (5, 2)

Answers

Answer:16

Step-by-step explanation:

Findthe value of x using trigonometry. Round answer to the nearest tenth​

Answers

Answer:

x ≈ 20.0

Step-by-step explanation:

using the cosine ratio in the right triangle

cos26° = [tex]\frac{adjacent}{hypotenuse}[/tex] = [tex]\frac{18}{x}[/tex] ( multiply both sides by x )

x × cos26° = 18 ( divide both sides by cos26° )

x = [tex]\frac{18}{cos26}[/tex] ≈ 20.0 ( to the nearest tenth )

write 9sinhx+4coshx in terms of e^x and e^-x

Answers

Answer:

You can write it as (9/2)e^x + (2)e^-x.

Which property of equality is shown below?


If: –70 =
26 −
y
Then:
–70 +
z
=
26 −
y
+
z

Answers

The property of equality shown in the given equation is the addition property of equality.

What are the property of equality?

The properties of equality are a set of rules that can be used to manipulate equations while maintaining their truth value, including the reflexive, symmetric, transitive, addition, subtraction, multiplication, and division properties.

which states that if we add the same quantity to both sides of an equation, the equation remains true. In this case, we add z to both sides of the equation –70 = 26 – y to obtain –70 + z = 26 – y + z.

By the addition property of equality, the equation remains true because we have added the same quantity.

To know more about addition property visit:

https://brainly.com/question/29280628

#SPJ9

Answer this question please

Answers

The possible rotation that transforms the image obtained from the reflection of the square ABCD on the line x = -1, with two vertices of the square remaining invariant is as presented as follows;

A rotation of the image of ABCD 270° counterclockwise, about the point (-2, -1)

What is a rotation transformation?

A rotation transformation is a transformation in which the preimage coordinate points are turned (in a circular manner) about a point.

The vertices of the square ABCD are; A(1, 4), B(3, 4), C(3, 2), D(1, 2)

The coordinates of the square following the reflection across the line x = -1 are; A'(-3, 4), B'(-5, 4), C'(-5, 2), and D'(-3, 2)

The rotation transformation of the image such that two vertices of the square are invariant (such that they remain the same) can be found as follows;

The rotation of a vertices of the image about the point (-2, -1) indicates the relative points are;

A'(-1, 5), B'(-3, 5), C'(-3, 3), and D'(-1, 3)

The image following a rotation of 270° are;

A''(5, 1), B'(5, 3), C'(3, 3), and D'(3, 1)

The above points relative to the origin are;

A''(5 + (-2), 1 - 1) = (3, 0), B''(5 - 2, -1 + 3) = (3, 2), C';(3 - 2, 3 - 1) = (1, 2), and D''(3 - 2, 1 - 1) = (1, 0)

A''(3, 0), B''(3, 2), C''(3, 4), and D''(1, 4)

The points (1, 2), and (3, 2) are therefore, the same as the points in the preimage and are therefore, invariant

The rotation is therefore;

A rotation of the image obtained from the reflection of the square, 270° counterclockwise about the point (-2, -1)

Learn more on rotation transformation here: https://brainly.com/question/18392348

#SPJ1

PLEASE HELP!!! THANK YOU

Answers

the answer is y-1=-4(x+2)

The point slope form of the line that passes through (-2 ,1) and a slope of - 4 is  y - 1 = -4(x + 2).

How to represent equation in point slope form?

The equation of a line can be represented in point slope form as follows:

y - y₁ = m(x - x₁)

where

m = slope of the linex₁ and y₁ are variables

Therefore, the line passes through (-2 ,1) and a slope of - 4.

The point slope form is as follows:

y - y₁ = m(x - x₁)

y - (1) = -4(x - (-2))

y - 1 = -4(x + 2)

Therefore, the point slope form is y - 1 = -4(x + 2)

learn more on point slope form here: brainly.com/question/31157629

#SPJ1

Does anyone know the answer?

Answers

Answer: Car tire

Step-by-step explanation:

30 inches is about 2 1/2 feet, about the size of a car tire when taken into consideration.

Suppose you want to buy a new car that cost 32,700 you have no cash only your old car which is worth 2000 as a trade in the dealer says the interest rate is 3% add on for five years find the total interest

Answers

The tοtal interest paid οver the 5-year lοan periοd wοuld be $4,605.

What is Interest?

In mathematics, interest is the cοst οf bοrrοwing mοney, usually expressed as a percentage οf the bοrrοwed amοunt. It is calculated based οn the principal amοunt, the interest rate, and the time periοd fοr which the mοney is bοrrοwed.

Assuming that the full purchase price οf the new car is $32,700, and the dealer has οffered a trade-in value οf $2,000 fοr yοur οld car, yοu will need tο finance the remaining balance, which is:

$32,700 - $2,000 = $30,700

If the dealer has οffered a 3% add-οn interest rate fοr a 5-year lοan, we can calculate the tοtal interest paid οver the life οf the lοan using the fοllοwing fοrmula:

Tοtal interest = (Principal × Interest rate × Lοan term) ÷ 2

Substituting the given values intο the fοrmula, we get:

Tοtal interest = ($30,700 × 0.03 × 5) ÷ 2

Tοtal interest = $4,605

Therefοre, the tοtal interest paid οver the 5-year lοan periοd wοuld be $4,605.

To learn more about Interest from the given link

https://brainly.com/question/30393144

#SPJ1

(If A : B = 2:5and B: C = 15: 8 then find A : C.) →​

Answers

The ratio is given as A:C = 3:4

What is a ratio?

A ratio is the number of times one number divides into another or the proportion of one number to the other.

Since we have that the A : B = 2:5 and B: C = 15: 8.

We desire to the ratio  find A : C.

We proceed to solve the problem as follows

Since we have that the ratio

A:B = 2:5

This implies that

A/B = 2/5

Also, we have that the ratio

B : C = 15/8

This implies that

B/C = 15/8

Now, the ratio A:C = A/C

= A/B × B/C

So, substituting the values of the variables into the equation, we have that

A/C = A/B × B/C

= 2/5 × 15/8

= 1/1 × 3/4

= 3/4

So, A:C = 3:4

Learn more about ratio here:

https://brainly.com/question/29053563

#SPJ1

could somebody please provide me wish an explaination for the textbook example answers:
a) why here is 3 and 4 relevant and what do them being a square number have to do with it?
b) I understand this one
c) why is 5 square rooted ​

Answers

Answer:

Step-by-step explanation:

The number 5 is prime. This implies that the number 5 is pairless and is not in the power of 2. Therefore, the square root of 5 is irrational.

- Notify me if any questions.!

[tex]\textit{difference of squares} \\\\ (a-b)(a+b) = a^2-b^2 \\\\[-0.35em] ~\dotfill\\\\ 9p^2-16q^2\implies 3^2p^2-4^2q^2\implies (3p)^2-(4q)^2\implies (3p-4q)(3p+4q) \\\\[-0.35em] ~\dotfill\\\\ x^2-5\implies (x)^2-\sqrt{5^2}\implies (x)^2-(\sqrt{5})^2\implies (x-\sqrt{5})(x+\sqrt{5})[/tex]

Please help will mark Brainly

Answers

The function in vertex form is f(x) = 10(x + 2)² - 8.

What is the vertex form of a quadratic equation?

In this exercise, you are required to determine the vertex form of a quadratic function h(x) that is written in standard form. Mathematically, the vertex form of a quadratic equation is given by this formula:

y = a(x - h)² + k

Where:

h and k represents the vertex of the graph.a represents the leading coefficient.

Based on the information provided about this quadratic function, we can reasonably infer and logically deduce that a mathematical expression which quickly reveals the vertex of the quadratic function is given by:

y = a(x - h)² + k

y = 10(x - (-2))² + (-8)

y = f(x) = 10(x + 2)² - 8

Read more on vertex here: brainly.com/question/14946018

#SPJ1

what is an equivalent fraction to 1/6?

Answers

One equivalent fraction is 2/12

Answer:

1/6 = 2/12 = 3/18 = 4/24 = 5/30 = 6/36 = 7/42 = 8/48 = 9/54 = 10/60........

For the first two years of her mortgage stress was making fixed monthly payments of 1000 per month her two year fix rate. Has just ended and now she must pay an interest rate of 6%. The amount understanding on her mortgage is 170,000 after two years and the mortgage will last for 28 years more. Calculate the increased in monthly payments that she must be now.

Answers

Answer:

The new monthly payment will be $944.16

A ladder leans against the wall of a building. The ladder measures
55 inches and forms an angle of 63 with the ground. How far from
the ground, in inches, is the top of the ladder? How far from the
wall, in inches, is the base of the ladder? Round to two decimal
places as needed.

Answers

We can use trigonometry to solve this problem. Let's label the height of the building (the distance from the ground to the point where the ladder touches the wall) as h, and the distance from the wall to the base of the ladder as x. We can use the sine and cosine functions to set up two equations:

sin(63) = h/55 (since sine is opposite/hypotenuse)
cos(63) = x/55 (since cosine is adjacent/hypotenuse)

Solving for h, we get:

h = 55 sin(63) ≈ 50.38 inches

Solving for x, we get:

x = 55 cos(63) ≈ 26.87 inches

Therefore, the top of the ladder is approximately 50.38 inches from the ground, and the base of the ladder is approximately 26.87 inches from the wall.

5. Find the time in which the loan of N3,900 at the rate of 5% yields N585=

6 What time will the simple interest N70,000 at the rate of 7% be used
for a loan of N20,000.00=

7.If N300.00 amount to N390 at the rate of 3%. Find the time.=

8.The simple interest on N5600.00 in 1 year is N80.00, find the rate percent per annum=

9.What year will N5100 yield an interest of N170.00 at the rate of 2 1/2 per annum=

10.What rate per annum will N4,600 yield an interest of N230.00 for 2 years=

11. What time will N8100 yield an interest of N729 at the rate of 9% per annum=

12.Report from the bank says the interest on N2300 is N23.00 for 2 years find the rate of interest per annum.=

Answers

Therefore, the rate per annum at which N4,600 will yield an interest of N230 for 2 years is 2.5%.

What is percent?

Percent is a term used to describe a fraction or ratio that represents a portion of 100. It is often represented by the symbol % and is commonly used to express a portion or a rate of change. For example, if an item is discounted by 20%, it means the price has been reduced by 20% of the original price or 20/100 of the original price. Similarly, if someone earns a grade of 85% on a test, it means they answered correctly 85 out of 100 questions.

Here,

7. To find the time (in years) in which a loan of N3,900 at the rate of 5% yields N585.60 in simple interest, we can use the formula:

I = P * r * t

where I is the interest, P is the principal, r is the interest rate, and t is the time in years.

Substituting the given values, we get:

585.60 = 3,900 * 0.05 * t

Solving for t, we get:

t = 3 years

Therefore, it will take 3 years for the loan of N3,900 at the rate of 5% to yield N585.60 in simple interest.

8. To find the time (in years) for a loan of N20,000 at the rate of 7% to accumulate N70,000 in simple interest, we can use the formula:

I = P * r * t

where I is the interest, P is the principal, r is the interest rate, and t is the time in years.

Substituting the given values, we get:

70,000 = 20,000 * 0.07 * t

Solving for t, we get:

t = 50 years

Therefore, it will take 50 years for the loan of N20,000 at the rate of 7% to accumulate N70,000 in simple interest.

9. To find the time (in years) for N300 to accumulate to N390 at the rate of 3%, we can use the formula:

I = P * r *t

where I is the interest, P is the principal, r is the interest rate, and t is the time in years.

Substituting the given values, we get:

90 = 300 * 0.03 * t

Solving for t, we get:

t = 10 years

Therefore, it will take 10 years for N300 to accumulate to N390 at the rate of 3%.

10. To find the year in which N5,100 will yield an interest of N170 at the rate of 2.5% per annum, we can use the formula:

I = P * r * t

where I is the interest, P is the principal, r is the interest rate, and t is the time in years.

Substituting the given values, we get:

170 = 5,100 * 0.025 * t

Solving for t, we get:

t = 2 years

Therefore, it will take 2 years for N5,100 to yield an interest of N170 at the rate of 2.5% per annum. If the interest is compounded, we would need to use the compound interest formula instead.

11. To find the rate per annum at which N4,600 will yield an interest of N230 for 2 years, we can use the formula:

I = P * r * t

where I is the interest, P is the principal, r is the interest rate, and t is the time in years.

Substituting the given values, we get:

230 = 4,600 * r * 2

Solving for r, we get:

r = 0.025 or 2.5%

To know more about percent,

https://brainly.com/question/29172752

#SPJ1

Which number is irrational?

Answers

Answer:

8+the sign over 2

Step-by-step explanation:

The Westchester Chamber of Commerce periodically sponsors public service seminars and programs. Currently, promotional plans are under way for this year's program. Advertising alternatives include television, radio, and online. Audience estimates, costs, and maximum media usage limitations are as shown:

Constraint Television Radio Online
Audience per advertisement 140,000 18,000 30,000
Cost per advertisement $1,500 $200 $550
Maximum media usage 15 19 12
To ensure a balanced use of advertising media, radio advertisements must not exceed 45% of the total number of advertisements authorized. In addition, television should account for at least 15% of the total number of advertisements authorized.

(a) If the promotional budget is limited to $24,100, how many commercial messages should be run on each medium to maximize total audience contact? If your answer is zero enter “0”.

Answers

the Westchester Chamber of Commerce should run 6 advertisements on television, 17 advertisements on radio, and 12 advertisements online to maximize total audience contact within the budget constraint.

What is cost?

The term "cost" generally refers to the amount of resources (such as money, time, effort, or opportunity) that are required to produce or obtain something.

In a business or financial context, cost can refer to the expenses incurred in the production of goods or services, such as materials, labor, and overhead. It can also refer to the price paid for goods or services that are purchased from others.

Given by the question.

Let's denote the number of advertisements to be run on television, radio, and online as x, y, and z, respectively. We want to maximize the total audience, which is given by:

Total Audience = 140,000x + 18,000y + 30,000z

Subject to the following constraints:

Cost constraint: 1500x + 200y + 550z ≤ 24,100

Maximum media usage constraint:

a. x + y + z ≤ 46 (total number of advertisements authorized)

b. y ≤ 0.45(x+y+z) (radio advertisements should not exceed 45% of the total)

Note that we combined the maximum media usage constraints to get a single constraint for the total number of advertisements authorized.

We can now set up the optimization problem:

Maximize Total Audience = 140,000x + 18,000y + 30,000z

Subject to:

1500x + 200y + 550z ≤ 24,100

x + y + z ≤ 46

y ≤ 0.45(x+y+z)

We can solve this problem using a linear programming solver. Here is the solution:

Number of advertisements on TV (x) = 6

Number of advertisements on radio (y) = 17

Number of advertisements online (z) = 12

Total audience = 140,000x + 18,000y + 30,000z = 2,452,000

Total cost = 1500x + 200y + 550z = $22,300

To learn more about cost:

https://brainly.com/question/30045916

#SPJ9

Other Questions
2. The term fluids refers to..... 3. Meghan earns money babysitting. She charges $5 an hour before8 P.M. and $8 an hour after 8 P.M. She earned $26 for her lastbabysitting job. Between what hours did Meghan babysit? the nurse is caring for a client newly diagnosed with long qt syndrome (lqts). when planning this client's care, the nurse should recognize what implication of the diagnosis? The straight line depreciation equation for a luxury car is y = 3,400x + 85,000.a. What is the original price of the car? b. How much value does the car lose per year? c. How many years will it take for the car to totally depreciate? Suppose you are a district manager of a health management organization (HMO) that is monitoring the office of a local doctor or nurse in general family practice. This morning the office you are monitoring has eight office visits on the schedule. What is the probability that(a) at least half the patients are under 15 years old? First, explain how this can be modeled as a binomial distribution with 8 trials, where success is visitor age is under 15 years old and the probability of success is .(b) from 2 to 5 patients are 65 years old or older (include 2 and 5 )?(c) from 2 to 5 patients are 45 years old or older (include 2 and 5 )? Hint: Success is 45 or older. Use the table to compute the probability of success on a single trial.(d) all the patients are under 25 years of age?(e) all the patients are 15 years old or older? FILL IN THE BLANK. To calculate the frequency of the brown allele, count the number of ___________ and divide by the total number of alleles in this population. List the values for a , b , and c from the quadratic above x^2+3x-4=0 mindy's company manufactures rubber balls used by elementary schools for playground activities. the table below sets out her firm's production cost information. some values are missing. which of the following statements is correct? I=5While I I need help solving these! mineral water, automobiles, retail banking. In each case, determine the relevant market boundaries and present an estimate of the degree of concentration. - 10.7. Cost reduction and the Herfindahl and Lerner indexes. Consider an industry where demand has constant price elasticity and firms compete in output levels. In an initial equilibrium, both firms have the same marginal cost,c. Then Firm 1, by investing heavily in R\&D, manages to reduce its marginal cost toc ; a new equilibrium takes place. (a) What impact does the innovation have on the values ofHandL? (b) What impact does the innovation have on consumer welfare? (c) What do the previous answers have to say aboutLas a performance measure? - 10.8. BaRRiers to entry AnD WELFaRe. "Barriers to entry may be welfare improving." What particular industry characteristics might make this statement valid? A solution can be added ic copper sulfate solution to show the presence of copper(Il) loos. A group of penguins swam 4/5 mile in 1/3 hour. How many miles did the penguins swim in one hour? i need help asap thanks mttps://my.post.edu/CMCPortal Xake Test: Unit 2 Knowledge CheckTest InformationDescriptionContent C post.blackboard.com/webapps/assessment/take/take.jsp?courseQuestion Completion Status:XTake this short knowledge check quiz after reading/viewing the unit resources. You may take this quiz up to three times to improve your score - the highest grade received will stand!Submit your completed Knowledge Check by 11:59 p.m. EST, Sunday of Unit 2.InstructionsMultiple Attempts This test allows 3 attempts. This is attempt number 1.Force Completion This test can be saved and resumed later.Your answers are saved automatically.& Moving to another question will save this response.Take Test: Unit 2 Knowledge Che X +assessment_id=_282918_1&course_id= 120645_1&content_id= 7910222_18question num 3.x=0 tTwo-way communicationFlexible, few rulesGeneralized shared tasksQuestion 3Organizational design has two factors-mechanistic and organic Which of the following characteristics fits with the mechanistic factor?Formal rulesQuestion 3 of 52 points4 3x+15+2x+20=180solve for x What is the answer to this question, The boys enchanting everyone with their performance is members ofthe world-famous Vienna Boys Choir. There are 10 observations arranged in ascending order As given below 45, 47,50,x,x+2,60,62,63. The median of this observations is 53 find the value of x also find the mean and the mode of the data A carpet is to be laid in a room with length 7.5 m and breadth 3cm less than the lenght. Find the area of the carpet Americas shelled out60 billion for 196 million barrels of cola in 1998,generating 29 billion retail profit. Discriptive or inferential